6
$\begingroup$

Does anyone recognize this problem? There are $2p$ equations and $2p$ unknowns, and it feels like a classic, but I've never encountered it before:

Given $d_1, d_2, \ldots d_p$, find $a_1, a_2, \ldots a_p$ and $n_1, n_2, \ldots n_p$: $$ \left[ \begin{array}{ccc} d_1\\ d_2\\ d_3\\ \vdots\\ d_{2p} \end{array} \right] = \left[ \begin{array}{ccc} a_1, ~a_2, \cdots ~a_p\\ a_1^2, ~a_2^2, \cdots ~a_p^2\\ a_1^3, ~a_2^3, \cdots ~a_p^3\\ \vdots\\ a_1^{2p}, ~a_2^{2p}, \cdots ~a_p^{2p} \end{array} \right] \cdot \left[ \begin{array}{c} n_1\\ n_2\\ \vdots\\ n_p \end{array} \right] $$

(and where $a \succeq 0$ and $n \succeq 0$ and only real values are considered, so wherever there can be positive and negative roots for the values of $a_i$, only the positive roots need be considered).

It seems straightforward to solve with elimination: From the first row, let $a_1 = \frac{d_1-\sum_{i>1} a_i n_i}{n_1}$. Then from the second row, solve for $a_2$ (which will now involve a quadratic, etc.). Because the polynomials get larger, solving it with elimination in this manner can be quite slow (I crashed sage doing this on a fairly small problem).

Has this family of problems been characterized well in the literature and can it be solved efficiently in practice? Even a numerical solution would suffice, as long as it will converge certainly (no multi-start Levenberg-Marquardt, etc.)...

$\endgroup$

4 Answers 4

10
$\begingroup$

Let $$f(X) = (X - a_1) \cdots (X - a_p) = X^p + e_1 X^{p-1} + \dots + e_p.$$ Using $f(a_i) = 0$ for each $i$, we get \begin{equation} \begin{bmatrix} d_{p+1} & d_p & \cdots & d_1 \\ d_{p+2} & d_{p+1} & \cdots & d_2 \\ \vdots & \vdots & \ddots & \vdots \\ d_{2p} & d_{2p-1} & \cdots & d_p \end{bmatrix} \begin{bmatrix} 1 \\ e_1 \\ \vdots \\ e_p \end{bmatrix} = \begin{bmatrix} 0 \\ 0 \\ \vdots \\ 0 \end{bmatrix} \end{equation} Note that the dimension of the first matrix is $p \times (p+1)$. In the general case, it will be possible to determine $e_1, \dots, e_p$ uniquely. Then calculate the roots of $f(X)$ to obtain $a_i$, and the remaining $n_i$ part will be a simple elimination problem.

If the dimension of the null space of the '$d_i$ matrix' is greater than $1$, it probably means that $a_i$ are not pairwise distinct. Then you might want to check whether system is also solvable with a smaller number of variables.

$\endgroup$
0
6
$\begingroup$

Actually, the problem precisely as you've stated it can't be solved: you can't write $(1,0,0,0)$ as a sum of multiples of two vectors $(a,a^2,a^3,a^4)$ and $(b,b^2,b^3,b^4)$, but this is easily fixed by starting with the zeroth power instead of the first. (Even more degenerately, $(1,0)$ isn't a multiple of $(a,a^2)$ for any $a$.)

Dongryul Kim's answer is basically correct, but let me expand a little bit on it: the span of the vectors $(1,a,\dots, a^{n-1})$ for $a\in A$ for some finite subset $A$ in any field has dimension equal to the size of $A$ if $\# A< n$, and spans the whole space otherwise (note how important it is that we start at the zeroth power to make this work). If $\# A< n$, then the orthogonal to this space is spanned by vectors of the form $(0,\dots,0,e_{\# A},\dots, e_1,1,0\dots,0)$ where $\prod_{a\in A}(X-a)=X^{\# A}+e_1X^{\# A-1}+\cdots+e_{\# A}$ (these are obviously orthogonal to the vectors, and have the correct dimensional span). Thus, if we want to write $(d_1,\dots, d_n)$ as a sum of these vectors, we have to find numbers $1,e_1,\dots,e_q$ such that $(d_1,\dots, d_n)$ is orthogonal to all of these vectors. That is, we want to consider the equation \begin{equation} \begin{bmatrix} d_{q+1} & d_{q} & \cdots & d_1 \\ d_{q+2} & d_{q+1} & \cdots & d_2 \\ \vdots & \vdots & \ddots & \vdots \\ d_{n} & d_{n-1} & \cdots & d_{n-q} \end{bmatrix} \begin{bmatrix} 1 \\ e_1 \\ \vdots \\ e_q \end{bmatrix} = \begin{bmatrix} 0 \\ 0 \\ \vdots \\ 0 \end{bmatrix}. \end{equation} For $q=1$, this will only have a non-trivial solution if all the $d_i$'s are 0. Otherwise, there will be a unique smallest integer $q>1$ such that this equation has a non-trivial solution. In that case, we have that $(d_1,\dots, d_n)$ is in the span of the vectors $(1,a,\dots, a^{n-1})$, for $a$ ranging over the roots of $X^{q}+e_1X^{q-1}+\cdots+e_q$, and one can easily solve for the $n_i$'s; by construction, we've seen that these roots are the unique minimal set such that $(d_1,\dots, d_n)$ is in the span. By basic linear algebra, we must have $q\leq \lceil n/2\rceil$, since at that point, the matrix has more columns than rows, and must have a non-trivial solution. Obviously, generically, we'll have equality there.

$\endgroup$
3
  • $\begingroup$ Thanks a lot for the description, I wouldn't have understood Dongryul Kim's answer without it. $\endgroup$
    – user
    Aug 6, 2015 at 2:57
  • $\begingroup$ Hi Ben, do you know if this same approach would work if the columns of the $A$ matrix were of the form $(1,a,a^2,a^4,a^8...)$ instead (but still with unique powers $0,1,2,4,8...$)? When I use $a_1 = 0.7, a_2 = 0.2$ and $n_1=2,n_2=1$, I can recover the $a_i$ values from $d_0,d_1,d_2,d_3$ ($e=(1,-0.15,0.005)$), but cannot seem to recover them from $d_0,d_1,d_2,d_4$ ($e=(1,0.665,-0.056125)$), even when I try inserting 0s into $e$ to correspond to the terms $X^4, X^2, X^1, X^0$. Does the approach require an evenly spaced sequence of powers or does it generalize to unique powers? Vielen Dank! $\endgroup$
    – user
    Aug 21, 2015 at 11:59
  • $\begingroup$ @user Seems unlikely; the important property evenly spaces powers have is that their orthogonal is easily described. $\endgroup$
    – Ben Webster
    Aug 24, 2015 at 19:07
2
$\begingroup$

Again not an answer, but maybe a useful attack line. Your system is a more general version of the ones to be solved to determine nodes and weights of the Gaussian quadrature. Indeed, suppose that $d_i = \int_{a}^b x^i \omega(x) \mathrm{d}x$ for each $i$, and for a certain weight function $\omega(x)$; then, you are looking for a set of nodes $a_i$ and weights $n_i$ such that the quadrature formula $\sum_{i=1}^p n_i f(a_i) \approx \int_{a}^b \omega(x) f(x) \mathrm{d}x$ is eaxct for all polynomials of degree at most $2p-1$.

Look for the derivation of the Gaussian weights and nodes; the theorems there might be generalized to your setting. If you only have one of these systems to solve, maybe you are lucky and it corresponds to a known classical weighting function (Hermite, Laguerre...).

$\endgroup$
1
  • $\begingroup$ This is actually very similar to the motivation I was using to approximate a more complex problem. Thanks for pointing me in the direction regarding quadrature. $\endgroup$
    – user
    Aug 6, 2015 at 3:00
1
$\begingroup$

Not an answer, but a suggestion. Eliminate the $n_i$'s. So let $A$ be the top $p$-by-$p$ part of your matrix and let $B$ be the bottom $p$-by-$p$ part, also let $b$ be the top $p$ entries of the $d$-vector and let $c$ be the bottom $p$ entries. Then you have $b=An$ and $c=Bn$, so $c=BA^{-1}b$. The next thing to observe is that $$ B = A\begin{pmatrix} a_1^p & 0 & \ldots & 0\\ 0 & a_2^p & \ldots & 0 \\ \vdots &&\cdots &\vdots \\ 0 & 0 &\ldots &a_p^p \end{pmatrix}. $$ So you get $p$ equations for the $a_1,\ldots,a_p$ that look like $$ c = A\begin{pmatrix} a_1^p & 0 & \ldots & 0\\ 0 & a_2^p & \ldots & 0 \\ \vdots &&\cdots &\vdots \\ 0 & 0 &\ldots &a_p^p \end{pmatrix} A^{-1} b. $$ The matrix $A$ is a vanderMonde matrix, and there are formulas (of sorts) for its inverse, for example in Macon, N.; A. Spitzbart. "Inverses of Vandermonde Matrices". The American Mathematical Monthly 65 (2): 95–100. https://www.jstor.org/stable/2308881 .

$\endgroup$
1
  • $\begingroup$ Thank you, I tried to find the name of this type of matrix. $\endgroup$
    – user
    Aug 6, 2015 at 3:00

Your Answer

By clicking “Post Your Answer”, you agree to our terms of service and acknowledge you have read our privacy policy.

Not the answer you're looking for? Browse other questions tagged or ask your own question.